(44 intermediate revisions by 2 users not shown)
Line 1: Line 1:
<br>
+
[[Category:ECE]]
 +
[[Category:QE]]
 +
[[Category:CNSIP]]
 +
[[Category:problem solving]]
 +
[[Category:automatic control]]
 +
[[Category:optimization]]
  
= [[ECE PhD Qualifying Exams|ECE Ph.D. Qualifying Exam]]: Automatic Control (AC)- Question 3, August 2011 =
+
= [[ECE PhD Qualifying Exams|ECE Ph.D. Qualifying Exam]] in "Automatic Control" (AC)  =
  
&nbsp;<font color="#ff0000"><span style="font-size: 19px;"><math>\color{blue}\text{5. } \left( \text{20 pts} \right) \text{ Consider the optimization problem, }</math></span></font>  
+
= [[ECE-QE_AC3-2011|Question 3, August 2011]], Part 5=
 +
 
 +
:[[ECE-QE AC3-2011 solusion-1|Part 1]],[[ECE-QE AC3-2011 solusion-2|2]],[[ECE-QE AC3-2011 solusion-3|3]],[[ECE-QE AC3-2011 solusion-4|4]],[[ECE-QE_AC3-2011_solusion-5|5]]
 +
 
 +
----
 +
 
 +
&nbsp;<font color="#ff0000"><span style="font-size: 19px;"><math>\color{blue}\text{5. } \left( \text{20 pts} \right) \text{ Consider the following optimization problem, }</math></span></font>  
  
 
<font color="#ff0000">&nbsp; &nbsp; &nbsp; &nbsp; &nbsp; &nbsp; &nbsp; &nbsp; &nbsp; &nbsp; &nbsp; &nbsp; &nbsp; &nbsp;&nbsp;</font><math>\text{optimize} \left(x_{1}-2\right)^{2}+\left(x_{2}-1\right)^{2}</math>  
 
<font color="#ff0000">&nbsp; &nbsp; &nbsp; &nbsp; &nbsp; &nbsp; &nbsp; &nbsp; &nbsp; &nbsp; &nbsp; &nbsp; &nbsp; &nbsp;&nbsp;</font><math>\text{optimize} \left(x_{1}-2\right)^{2}+\left(x_{2}-1\right)^{2}</math>  
Line 9: Line 20:
 
&nbsp; &nbsp; &nbsp; &nbsp; &nbsp; &nbsp; &nbsp; &nbsp; &nbsp; &nbsp; &nbsp; &nbsp;&nbsp;<math>\text{subject to  }  x_{2}- x_{1}^{2}\geq0</math>  
 
&nbsp; &nbsp; &nbsp; &nbsp; &nbsp; &nbsp; &nbsp; &nbsp; &nbsp; &nbsp; &nbsp; &nbsp;&nbsp;<math>\text{subject to  }  x_{2}- x_{1}^{2}\geq0</math>  
  
&nbsp; &nbsp; &nbsp; &nbsp; &nbsp; &nbsp; &nbsp; &nbsp; &nbsp; &nbsp; &nbsp; &nbsp; &nbsp; &nbsp; &nbsp; &nbsp; &nbsp; &nbsp; &nbsp; &nbsp; &nbsp; &nbsp; &nbsp; &nbsp; &nbsp;<math>2-x_{1}-x_{2}\geq0, x_{1}\geq0.</math>  
+
&nbsp; &nbsp; &nbsp; &nbsp; &nbsp; &nbsp; &nbsp; &nbsp; &nbsp; &nbsp; &nbsp; &nbsp; &nbsp; &nbsp; &nbsp; &nbsp; &nbsp; &nbsp; &nbsp; &nbsp; &nbsp; &nbsp; &nbsp; &nbsp; &nbsp;<math>2-x_{1}-x_{2}\geq0</math>
 +
 
 +
&nbsp; &nbsp; &nbsp; &nbsp; &nbsp; &nbsp; &nbsp; &nbsp; &nbsp; &nbsp; &nbsp; &nbsp; &nbsp; &nbsp; &nbsp; &nbsp; &nbsp; &nbsp; &nbsp; &nbsp; &nbsp; &nbsp; &nbsp; &nbsp; &nbsp;<math>x_{1}\geq0.</math>  
  
 
<math>\color{blue} \text{The point }  x^{*}=\begin{bmatrix}
 
<math>\color{blue} \text{The point }  x^{*}=\begin{bmatrix}
 
0 & 0  
 
0 & 0  
\end{bmatrix}^{T} \text{ satisfies the KKT conditions.}</math>  
+
\end{bmatrix}^{T} \text{ satisfies the KKT conditions.}</math><br>  
  
<math>\color{blue}\left( \text{i} \right) \text{Does } x^{*} \text{ satisfy the FONC for minimum or maximum? Where are the KKT multipliers?}</math>  
+
----
 +
 
 +
'''Theorem:'''
 +
 
 +
For the problem: &nbsp; &nbsp;minimize &nbsp;<math>f \left( x \right)</math>
 +
 
 +
&nbsp; &nbsp; &nbsp; &nbsp; &nbsp; &nbsp; &nbsp; &nbsp; &nbsp; &nbsp; &nbsp; &nbsp; &nbsp; &nbsp; &nbsp;subject to &nbsp;&nbsp;<math>h \left( x \right) =0</math>
 +
 
 +
&nbsp; &nbsp; &nbsp; &nbsp; &nbsp; &nbsp; &nbsp; &nbsp; &nbsp; &nbsp; &nbsp; &nbsp; &nbsp; &nbsp; &nbsp; &nbsp; &nbsp; &nbsp; &nbsp; &nbsp; &nbsp; &nbsp; &nbsp; &nbsp;&nbsp;<math>g \left( x \right) \leq 0</math>
 +
 
 +
The '''KKT condition (FONC) '''for local minimizer&nbsp;<span class="texhtml">''x''<sup> * </sup></span>&nbsp;of <span class="texhtml">''f''</span>&nbsp;is:
 +
 
 +
<font color="#ff0000"><span style="font-size: 17px;">'''&nbsp; &nbsp; &nbsp;&nbsp;<math>\text{1. } \mu^{*}\geq0</math>'''</span></font>
 +
 
 +
<font color="#ff0000"><span style="font-size: 20px;">'''&nbsp; &nbsp; &nbsp;<math>\text{2. } Df\left ( x^{*} \right )+\lambda ^{*T}Dh\left ( x^{*} \right )+\mu ^{*T}Dg\left ( x^{*} \right )=0^{T}</math>'''</span></font>
 +
 
 +
<font color="#ff0000">&nbsp; &nbsp; &nbsp; &nbsp;&nbsp;<math>\text{3. } \mu ^{*T}g\left ( x^{*} \right )=0</math><br></font>
 +
 
 +
<font color="#ff0000"></font>&nbsp; &nbsp; &nbsp; &nbsp;&nbsp;<math>\text{4. } h\left ( x^{*} \right )=0</math>
 +
 
 +
&nbsp; &nbsp; &nbsp; &nbsp;&nbsp;<math>\text{5. } g \left( x^{*}  \right) \leq0</math><br>
 +
 
 +
'''Definision: Regular point''' &nbsp; &nbsp; &nbsp;&nbsp; &nbsp; &nbsp; &nbsp;&nbsp;
 +
 
 +
&nbsp; &nbsp; &nbsp; &nbsp; &nbsp;<math>x^{*} \text{ satisfy } h\left( x^{*} \right)=0, g\left( x^{*} \right)\leq0 \text{ and let } J\left(x^{*}\right)= \left \{  j:g_{j}\left(x^{*}\right)=0 \right \}</math>  
 +
 
 +
'''&nbsp; &nbsp; &nbsp; &nbsp; &nbsp;<math>x^{*}\text{ is regular point if } \nabla h_{i} \left( x^{*} \right),  \nabla g_{j} \left( x^{*} \right),  1\leq i\leq m, j\in J \left( x^{*} \right)</math> '''
 +
 
 +
'''SONC: '''Suppose that&nbsp;<span class="texhtml">''x''<sup> * </sup></span>&nbsp;is regular<br> &nbsp; &nbsp; &nbsp; &nbsp;&nbsp;<math>\text{1. } \mu ^{*}\geq0 \text{, } Df\left ( x^{*} \right )+\lambda ^{*T}Dh\left ( x^{*} \right )+\mu ^{*T}Dg\left ( x^{*} \right )=0^{T} \text{, } \mu ^{*T}g\left ( x^{*} \right )=0</math>
 +
 
 +
&nbsp; &nbsp; &nbsp; &nbsp;&nbsp;<math>\text{2. For all } y\in T\left( x^{*} \right ) \text{, we have } y^{T}L\left ( x^{\ast },\mu ^{\ast }, \lambda ^{\ast }\right )y\geq 0</math>
 +
 
 +
&nbsp; &nbsp; &nbsp; &nbsp; &nbsp; &nbsp; &nbsp; &nbsp;<math>T\left( x^{* } \right)= \left \{ y\in\Re^{n}: Dh\left( x^{*} \right)y=0, Dg_{j}\left( x^{*} \right)y=0, j\in J\left( x^{*} \right)  \right \}</math>
 +
 
 +
&nbsp; &nbsp; &nbsp; &nbsp; &nbsp; &nbsp; &nbsp; &nbsp;<math>J\left(x^{*}\right)= \left \{  j:g_{j}\left(x^{*}\right)=0 \right \}</math>
 +
 
 +
'''SOSC:''' There exist a feasible point&nbsp;<span class="texhtml">''x''<sup> *&nbsp;<sub></sub><sub></sub></sup></span>that&nbsp;
 +
 
 +
&nbsp; &nbsp; &nbsp; &nbsp;&nbsp;<math>\text{1. } \mu ^{*}\geq0 \text{, } Df\left ( x^{*} \right )+\lambda ^{*T}Dh\left ( x^{*} \right )+\mu ^{*T}Dg\left ( x^{*} \right )=0^{T} \text{, } \mu ^{*T}g\left ( x^{*} \right )=0</math>
 +
 
 +
&nbsp; &nbsp; &nbsp; &nbsp;&nbsp;<math>\text{2. For all } y\in \tilde{T}\left( x^{* }\mu^{*} \right) \text{, we have } y^{T}L\left ( x^{\ast },\mu ^{\ast }, \lambda ^{\ast }\right )y\geq 0</math>
 +
 
 +
&nbsp; &nbsp; &nbsp; &nbsp; &nbsp; &nbsp; &nbsp; &nbsp;<math>\tilde{T}\left( x^{* },\mu^{*} \right)= \left \{ y: Dh\left( x^{*} \right)y=0, Dg_{i}\left( x^{*} \right)y=0, i\in \tilde{J}\left( x^{*},\mu^{*} \right)  \right \}</math>
 +
 
 +
&nbsp; &nbsp; &nbsp; &nbsp; &nbsp; &nbsp; &nbsp;&nbsp;<math>\tilde{J}\left ( x^{\ast },\mu ^{\ast } \right )= \left \{ i:g_{i}\left ( x^{\ast } \right ) = 0,\mu_{i}^{\ast }> 0\right \}</math>
 +
 
 +
'''Process:'''
 +
 
 +
&nbsp; &nbsp; &nbsp; a. Write down the KKT condition for this probelm
 +
 
 +
&nbsp; &nbsp; &nbsp; b. Find all points (and KKT multipliers) satisfying the KKT condition. In each case, determine if the point is regular.
 +
 
 +
&nbsp; &nbsp; &nbsp; c. Find all points in part b that also satisfy the SONC.
 +
 
 +
&nbsp; &nbsp; &nbsp; d. Find all points in part c that also satisfy the SOSC.
 +
 
 +
&nbsp; &nbsp; &nbsp; e. Find all points in part c that are local minimizers.
  
 
----
 
----
 +
 +
<math>\color{blue}\left( \text{i} \right) \text{Does } x^{*} \text{ satisfy the FONC for minimum or maximum? Where are the KKT multipliers?}</math>
  
 
<math>\color{blue}\text{Solution 1:}</math>  
 
<math>\color{blue}\text{Solution 1:}</math>  
  
<math>f\left( x \right) = \left(x_{1}-2\right)^{2}+\left(x_{2}-1\right)^{2}</math>  
+
&nbsp; &nbsp; &nbsp; &nbsp;&nbsp;<math>f\left( x \right) = \left(x_{1}-2\right)^{2}+\left(x_{2}-1\right)^{2}</math>  
  
<math>g_{1}\left( x \right) x_{1}^{2}-x_{2}\leq0</math>  
+
&nbsp; &nbsp; &nbsp; &nbsp;&nbsp;<math>g_{1}\left( x \right)=x_{1}^{2}-x_{2}</math>  
  
<math>g_{2}\left( x \right) x_{1}+x_{2}-2\leq0</math>  
+
&nbsp; &nbsp; &nbsp; &nbsp;&nbsp;<math>g_{2}\left( x \right)= x_{1}+x_{2}-2</math>  
  
<math>g_{3}\left( x \right) -x_{1}\leq0</math>  
+
&nbsp; &nbsp; &nbsp; &nbsp;&nbsp;<math>g_{3}\left( x \right)= -x_{1}</math>  
  
The problem is to optimize f(x), &nbsp;<math>\text{subject to } g_{1}\leq 0, g_{2}\leq 0, g_{3}\leq 0</math>  
+
&nbsp;<math>\text{ The problem is to optimize f(x), subject to } g_{1}\leq 0, g_{2}\leq 0, g_{3}\leq 0</math>  
  
<math>l\left( \mu ,\lambda \right)=\nabla f\left(x  \right)+\mu_{1} \nabla g_{1}\left( x \right)+\mu_{2} \nabla g_{2}\left( x \right)+\mu_{3} \nabla g_{3}\left( x \right)</math>  
+
<math>\text{Let }  l\left( \mu ,\lambda \right)=\nabla f\left(x  \right)+\mu_{1} \nabla g_{1}\left( x \right)+\mu_{2} \nabla g_{2}\left( x \right)+\mu_{3} \nabla g_{3}\left( x \right)</math>  
  
<font color="#ff0000"><span style="font-size: 17px;">'''&nbsp; &nbsp; &nbsp; &nbsp; &nbsp; &nbsp; &nbsp;&nbsp;<math>=\begin{pmatrix}
+
<font color="#ff0000"><span style="font-size: 17px;">'''&nbsp; &nbsp; &nbsp; &nbsp; &nbsp; &nbsp; &nbsp; &nbsp; &nbsp; &nbsp; &nbsp;&nbsp;<math>=\begin{pmatrix}
 
2x_{1}-4\\  
 
2x_{1}-4\\  
 
2x_{2}-2
 
2x_{2}-2
Line 50: Line 121:
 
<math>\mu_{1} g_{1}\left( x \right)+\mu_{2} g_{2}\left( x \right)+\mu_{3} g_{3}\left( x \right)</math>  
 
<math>\mu_{1} g_{1}\left( x \right)+\mu_{2} g_{2}\left( x \right)+\mu_{3} g_{3}\left( x \right)</math>  
  
&nbsp; &nbsp; &nbsp; &nbsp; &nbsp;<math>= \mu_{1} \left( x_{1}^2-x_{2} \right)+\mu_{2} \left( x_{1}+x_{2}-2 \right)+\mu_{3} \left( -x_{1} \right) =0</math><br>  
+
&nbsp; &nbsp; &nbsp; &nbsp; &nbsp; &nbsp;&nbsp;<math>= \mu_{1} \left( x_{1}^2-x_{2} \right)+\mu_{2} \left( x_{1}+x_{2}-2 \right)+\mu_{3} \left( -x_{1} \right) =0</math><br>  
  
 
<math>\text{Let } x^{*}=\begin{bmatrix}
 
<math>\text{Let } x^{*}=\begin{bmatrix}
Line 57: Line 128:
 
\end{bmatrix} \text{, }</math><br>  
 
\end{bmatrix} \text{, }</math><br>  
  
<math>\left\{\begin{matrix}
+
&nbsp; &nbsp; &nbsp; &nbsp; &nbsp; &nbsp;&nbsp;<math>\left\{\begin{matrix}
 
\nabla l\left( x,\mu \right)=\begin{pmatrix}
 
\nabla l\left( x,\mu \right)=\begin{pmatrix}
 
-4+\mu_{2}-\mu_{3}\\  
 
-4+\mu_{2}-\mu_{3}\\  
Line 79: Line 150:
 
<math>\text{ Standard form: optimize} \left(x_{1}-2\right)^{2}+\left(x_{2}-1\right)^{2}</math><br>  
 
<math>\text{ Standard form: optimize} \left(x_{1}-2\right)^{2}+\left(x_{2}-1\right)^{2}</math><br>  
  
&nbsp; &nbsp; &nbsp; &nbsp; &nbsp; &nbsp; &nbsp; &nbsp; &nbsp; &nbsp; &nbsp; &nbsp; &nbsp; &nbsp; &nbsp; &nbsp; &nbsp; <math>\text{subject to  }  g_{1}\left( x \right) x_{1}^{2}-x_{2}\leq0</math>  
+
&nbsp; &nbsp; &nbsp; &nbsp; &nbsp; &nbsp; &nbsp; &nbsp; &nbsp; &nbsp; &nbsp; &nbsp; &nbsp; &nbsp; &nbsp; &nbsp; &nbsp; <math>\text{subject to  }  g_{1}\left( x \right)= x_{1}^{2}-x_{2}\leq0</math>  
  
&nbsp; &nbsp; &nbsp; &nbsp; &nbsp; &nbsp; &nbsp; &nbsp; &nbsp; &nbsp; &nbsp; &nbsp; &nbsp; &nbsp; &nbsp; &nbsp; &nbsp; &nbsp; &nbsp; &nbsp; &nbsp; &nbsp; &nbsp; &nbsp; &nbsp; &nbsp; &nbsp; &nbsp; &nbsp; &nbsp;<math>g_{2}\left( x \right) x_{1}+x_{2}-2\leq0</math>  
+
&nbsp; &nbsp; &nbsp; &nbsp; &nbsp; &nbsp; &nbsp; &nbsp; &nbsp; &nbsp; &nbsp; &nbsp; &nbsp; &nbsp; &nbsp; &nbsp; &nbsp; &nbsp; &nbsp; &nbsp; &nbsp; &nbsp; &nbsp; &nbsp; &nbsp; &nbsp; &nbsp; &nbsp; &nbsp; &nbsp;<math>g_{2}\left( x \right)= x_{1}+x_{2}-2\leq0</math>  
  
&nbsp; &nbsp; &nbsp; &nbsp; &nbsp; &nbsp; &nbsp; &nbsp; &nbsp; &nbsp; &nbsp; &nbsp; &nbsp; &nbsp; &nbsp; &nbsp; &nbsp; &nbsp; &nbsp; &nbsp; &nbsp; &nbsp; &nbsp; &nbsp; &nbsp; &nbsp; &nbsp; &nbsp; &nbsp; &nbsp;<math>g_{3}\left( x \right) -x_{1}\leq0</math>  
+
&nbsp; &nbsp; &nbsp; &nbsp; &nbsp; &nbsp; &nbsp; &nbsp; &nbsp; &nbsp; &nbsp; &nbsp; &nbsp; &nbsp; &nbsp; &nbsp; &nbsp; &nbsp; &nbsp; &nbsp; &nbsp; &nbsp; &nbsp; &nbsp; &nbsp; &nbsp; &nbsp; &nbsp; &nbsp; &nbsp;<math>g_{3}\left( x \right)= -x_{1}\leq0</math>  
  
 
<math>\text{KKT condition: (1) } Dl\left( \mu ,\lambda \right)=Df\left(x  \right)+\mu_{1}Dg_{1}\left( x \right)+\mu_{2}Dg_{2}\left( x \right)+\mu_{3}Dg_{3}\left( x \right)</math><br>  
 
<math>\text{KKT condition: (1) } Dl\left( \mu ,\lambda \right)=Df\left(x  \right)+\mu_{1}Dg_{1}\left( x \right)+\mu_{2}Dg_{2}\left( x \right)+\mu_{3}Dg_{3}\left( x \right)</math><br>  
  
&nbsp; &nbsp; &nbsp; &nbsp; &nbsp; &nbsp; &nbsp; &nbsp; &nbsp; &nbsp; &nbsp; &nbsp; &nbsp; &nbsp; &nbsp; &nbsp; &nbsp; &nbsp; &nbsp; &nbsp; &nbsp; &nbsp; &nbsp; &nbsp; &nbsp; &nbsp; &nbsp; &nbsp; &nbsp; &nbsp; &nbsp; &nbsp; &nbsp; &nbsp; &nbsp;&nbsp;<math>=\left [ 2x_{1}-4+2\mu_{1}x_{1}+\mu_{2}-\mu_{3}, 2x_{2}-2-\mu_{1}+\mu_{2} \right ]</math>  
+
&nbsp; &nbsp; &nbsp; &nbsp; &nbsp; &nbsp; &nbsp; &nbsp; &nbsp; &nbsp; &nbsp; &nbsp; &nbsp; &nbsp; &nbsp; &nbsp; &nbsp; &nbsp; &nbsp; &nbsp; &nbsp; &nbsp; &nbsp;<math>=\left [ 2x_{1}-4+2\mu_{1}x_{1}+\mu_{2}-\mu_{3}, 2x_{2}-2-\mu_{1}+\mu_{2} \right ]=0</math>&nbsp;&nbsp; &nbsp; &nbsp; &nbsp; &nbsp; &nbsp; &nbsp; &nbsp; &nbsp; &nbsp; &nbsp; &nbsp; &nbsp; &nbsp; &nbsp; &nbsp; &nbsp; &nbsp; &nbsp; &nbsp; &nbsp; &nbsp;<math>\left ( 2 \right ) \mu^{T}g\left ( x \right )=0 \Rightarrow  \mu_{1}\left ( x_{1}^2-x_{2} \right )+\mu_{2}\left ( x_{1}+x_{2}-2 \right ) - \mu_{3}x_{1}=0</math>  
  
&nbsp; &nbsp; &nbsp; &nbsp; &nbsp; &nbsp; &nbsp; &nbsp; &nbsp; &nbsp; &nbsp; &nbsp; &nbsp; &nbsp; &nbsp; &nbsp; &nbsp; &nbsp; &nbsp; &nbsp; <math>\left ( 2 \right ) \mu^{T}g\left ( x \right )=0 \Rightarrow  \mu_{1}\left ( x_{2}^2-x_{2} \right )+\mu_{2}\left ( x_{1}+x_{2}-2 \right ) - \mu_{3}x_{1}=0</math>  
+
&nbsp; &nbsp; &nbsp; &nbsp; &nbsp; &nbsp; &nbsp; &nbsp; &nbsp; &nbsp; &nbsp; &nbsp; &nbsp; &nbsp; &nbsp; &nbsp; &nbsp;<math>\left ( 3 \right ) \mu_{1},\mu_{2},\mu_{3}\geq 0 \text{ for minimizer}</math><br>  
  
&nbsp; &nbsp; &nbsp; &nbsp; &nbsp; &nbsp; &nbsp; &nbsp; &nbsp; &nbsp; &nbsp; &nbsp; &nbsp; &nbsp; &nbsp; &nbsp; &nbsp; &nbsp; &nbsp; &nbsp;&nbsp;<math>\left ( 3 \right ) \mu_{1},\mu_{2},\mu_{3}\geq 0 \text{ for minimizer}</math><br>
+
&nbsp; &nbsp; &nbsp; &nbsp; &nbsp; &nbsp; &nbsp; &nbsp; &nbsp; &nbsp; &nbsp; &nbsp; &nbsp; &nbsp; &nbsp; &nbsp; &nbsp; &nbsp; &nbsp; &nbsp;&nbsp;<math>\mu_{1},\mu_{2},\mu_{3}\leq 0 \text{ for maximizer}</math>  
 
+
&nbsp; &nbsp; &nbsp; &nbsp; &nbsp; &nbsp; &nbsp; &nbsp; &nbsp; &nbsp; &nbsp; &nbsp; &nbsp; &nbsp; &nbsp; &nbsp; &nbsp; &nbsp; &nbsp; &nbsp; &nbsp; &nbsp; &nbsp; &nbsp;<math>\mu_{1},\mu_{2},\mu_{3}\leq 0 \text{ for maximizer}</math>  
+
  
 
&nbsp; &nbsp; &nbsp; &nbsp; &nbsp; &nbsp; &nbsp; &nbsp; &nbsp; &nbsp; &nbsp; &nbsp; &nbsp; &nbsp; &nbsp; &nbsp; &nbsp; &nbsp; &nbsp; &nbsp; <math>\text{where } \mu^{*}=\begin{bmatrix}
 
&nbsp; &nbsp; &nbsp; &nbsp; &nbsp; &nbsp; &nbsp; &nbsp; &nbsp; &nbsp; &nbsp; &nbsp; &nbsp; &nbsp; &nbsp; &nbsp; &nbsp; &nbsp; &nbsp; &nbsp; <math>\text{where } \mu^{*}=\begin{bmatrix}
Line 107: Line 176:
 
\nabla l\left( x,\mu \right)=\begin{pmatrix}
 
\nabla l\left( x,\mu \right)=\begin{pmatrix}
 
-4+\mu_{2}-\mu_{3}\\  
 
-4+\mu_{2}-\mu_{3}\\  
-2-\mu_{1}-\mu_{2}
+
-2-\mu_{1}+\mu_{2}
 
\end{pmatrix}=\begin{pmatrix} 0\\0 \end{pmatrix}\\  
 
\end{pmatrix}=\begin{pmatrix} 0\\0 \end{pmatrix}\\  
 
-2\mu_{2}=0
 
-2\mu_{2}=0
Line 135: Line 204:
 
\end{pmatrix}</math><br>  
 
\end{pmatrix}</math><br>  
  
<math>\tilde{T}\left( x^{* }\mu^{*} \right)= \left\{
+
<font color="#ff0000">'''<math>\tilde{T}\left( x^{* }\mu^{*} \right) : \left\{ \begin{matrix} y^{T}\binom{0}{-1} =0 \\ y^{T}\binom{-1}{0} =0 \end{matrix} \right. \Rightarrow \tilde{T}\left( x^{* }\mu^{*} \right)= \left \{ \binom{0}{0} \right \}</math><br>'''</font>  
\begin{matrix}
+
y^{T}\binom{0}{-1} =0 \\  
+
y^{T}\binom{-1}{0} =0  
+
\end{matrix}
+
\right. \Rightarrow \tilde{T}\left( x^{* }\mu^{*} \right)= \left \{ \binom{0}{0} \right \}</math><br>  
+
  
 
SOSC is trivially satisfied.  
 
SOSC is trivially satisfied.  
 +
 +
<math>\color{red} \text{This solution misunderstood the range of } y \text{ for SOSC condition } y^{T}L\left ( x^{\ast },\mu ^{\ast } \right )y\geq 0</math><br>
 +
 +
<math>\color{red} y\in \tilde{T}\left( x^{* }\mu^{*} \right) \text{, where } \tilde{T}\left( x^{* },\mu^{*} \right)= \left \{ y:Dg_{i}\left( x^{*} \right)y=0, i\in \tilde{J}\left( x^{*},\mu^{*} \right)  \right \}</math>
 +
 +
<math>\color{red} \tilde{J}\left ( x^{\ast },\mu ^{\ast } \right ) \text{ has constrain for } \mu ^{\ast } \text{, }  \tilde{J}\left ( x^{\ast },\mu ^{\ast } \right )= \left \{ i:g_{i}\left ( x^{\ast } \right ) = 0,\mu_{i}^{\ast }> 0\right \}</math>
 +
 +
<math>\color{red} \tilde{J}\left ( x^{\ast },\mu ^{\ast } \right ) \subset 
 +
J\left(x^{*}\right)</math>, &nbsp; &nbsp;<math>\color{red} T\left( x^{* } \right) \subset \tilde{T}\left( x^{* },\mu^{*} \right)</math>
  
 
----
 
----
Line 148: Line 221:
 
<math>\color{blue}\text{Solution 2:}</math>  
 
<math>\color{blue}\text{Solution 2:}</math>  
  
<font color="#ff0000"><span style="font-size: 17px;">'''<math>L\left ( x_{1}\mu \right )= D^{2} l \left ( x _{1}\mu \right )= \begin{bmatrix} 2+2\mu_{1} & 0 \\ 0 & 2 \end{bmatrix}</math>'''</span></font>  
+
<font color="#ff0000"><span style="font-size: 17px;">'''<math>L\left ( x_{1},\mu \right )= D^{2} l \left ( x _{1},\mu \right )= \begin{bmatrix} 2+2\mu_{1} & 0 \\ 0 & 2 \end{bmatrix}</math>'''</span></font>  
  
<math>\text{for point } x^{*}=\begin{bmatrix} 0 \\ 0 \end{bmatrix} \text{, we get } \mu_{1}=-2  \text{ from KKT condition.}</math>  
+
&nbsp; &nbsp; &nbsp; &nbsp; &nbsp; &nbsp; &nbsp; &nbsp; &nbsp; &nbsp;<math>\text{for point } x^{*}=\begin{bmatrix} 0 \\ 0 \end{bmatrix} \text{, we get } \mu_{1}=-2  \text{ from KKT condition.}</math>  
  
<font color="#ff0000" size="5">'''<math>\therefore L \left ( x^{*}, \mu ^{*}\right )=\begin{bmatrix} -2 & 0 \\ 0 & 2 \end{bmatrix}</math><br>'''</font>  
+
<font color="#ff0000" size="5">'''&nbsp; &nbsp; &nbsp; &nbsp; &nbsp; &nbsp;<math>\therefore L \left ( x^{*}, \mu ^{*}\right )=\begin{bmatrix} -2 & 0 \\ 0 & 2 \end{bmatrix}</math><br>'''</font>  
  
<font color="#ff0000"><span style="font-size: 20px;">'''<math>\tilde{T}\left( x^{* }\mu^{*} \right)= \left \{ y:Dg_{i}\left( x^{*} \right)y=0, i\in \tilde{J}\left( x^{*},\mu^{*} \right)  \right \}</math>'''</span></font>  
+
<font color="#ff0000"><span style="font-size: 20px;">'''<math>\tilde{T}\left( x^{* },\mu^{*} \right)= \left \{ y:Dg_{i}\left( x^{*} \right)y=0, i\in \tilde{J}\left( x^{*},\mu^{*} \right)  \right \}</math>'''</span></font>  
  
<math>\tilde{J}\left ( x^{\ast },\mu ^{\ast } \right )= \left \{ i:g_{i}\left ( x^{\ast } \right ) = 0,\mu_{i^{\ast }}> 0\right \}</math>&nbsp; &nbsp; &nbsp;<math>\therefore i= 2</math>  
+
&nbsp; &nbsp; &nbsp; &nbsp; &nbsp; &nbsp; &nbsp;<math>\tilde{J}\left ( x^{\ast },\mu ^{\ast } \right )= \left \{ i:g_{i}\left ( x^{\ast } \right ) = 0,\mu_{i}^{\ast }> 0\right \}</math>&nbsp; &nbsp; &nbsp;<math>\therefore i= 2</math>  
  
<math>\therefore \tilde{T}\left ( x^{\ast },\mu ^{\ast } \right )= \left \{ y:\left [ 1,1 \right ]y= 0 \right \}= \left \{ y:y_{1}= -y_{2} \right \}</math>  
+
&nbsp; &nbsp; &nbsp; &nbsp; &nbsp; &nbsp; &nbsp;<math>\therefore \tilde{T}\left ( x^{\ast },\mu ^{\ast } \right )= \left \{ y:\left [ 1,1 \right ]y= 0 \right \}= \left \{ y:y_{1}= -y_{2} \right \}</math>  
  
<math>\left [ y_{1},y_{2} \right ]\begin{bmatrix}
+
<math>y^{T}L\left ( x^{\ast },\mu ^{\ast } \right )y=\begin{bmatrix}
 +
y_{1}&
 +
y_{2}
 +
\end{bmatrix}\begin{bmatrix}
 
-2 & 0\\  
 
-2 & 0\\  
 
0 & 2
 
0 & 2
Line 168: Line 244:
 
\end{bmatrix} \geqslant 0</math>  
 
\end{bmatrix} \geqslant 0</math>  
  
<math>-2y_{1}^{2}+2y_{2}^{2}\geqslant 0\cdots \left ( 1 \right )</math>  
+
&nbsp; &nbsp; &nbsp; &nbsp; &nbsp; &nbsp; &nbsp; &nbsp; &nbsp; &nbsp;&nbsp;<math>-2y_{1}^{2}+2y_{2}^{2}\geqslant 0\cdots \left ( 1 \right )</math>  
  
<span class="texhtml">for ''y''<sub>1</sub> = ''y''<sub>2</sub>, (1) is always satisfied.</span>  
+
<span class="texhtml">&nbsp; &nbsp; &nbsp; &nbsp; &nbsp; &nbsp; &nbsp; &nbsp; &nbsp; &nbsp; for ''y''<sub>1</sub> = -&nbsp;''y''<sub>2</sub>, &nbsp;(1) is always satisfied.</span>  
  
<math>\therefore \text{For all } y\in T\left( x^{*} \right ) \text{, we have } y^{T}L\left ( x^{\ast },\mu ^{\ast } \right )y\geq 0</math>  
+
<math>\therefore \text{For all } y\in \tilde{T}\left( x^{* },\mu^{*} \right) \text{, we have } y^{T}L\left ( x^{\ast },\mu ^{\ast } \right )y\geq 0</math>  
  
<math>\therefore \text{point } x^{*} \text{satisfy the SOSC}</math>  
+
<math>\therefore \text{point } x^{*} \text{satisfy the SOSC}</math><br>
 +
 
 +
----
 +
----
 +
<math>\color{blue}\text{Related Problem: }</math>
 +
 
 +
<span class="texhtml">''&nbsp; &nbsp; &nbsp; &nbsp; &nbsp; &nbsp; &nbsp; &nbsp; &nbsp; minimize'''&nbsp; &nbsp;'''''<b>&nbsp;− ''x''<sub>2</sub> + (''x''<sub>1</sub> − 1)<sup>2</sup> − 2</b></span>
 +
 
 +
&nbsp; &nbsp; &nbsp; &nbsp; &nbsp; &nbsp; &nbsp; &nbsp; &nbsp;&nbsp;<math>\text{subject to } x_{1}+x_{2}\leq2</math>
 +
 
 +
<math>\text{KKT condition: 1. } \mu\geq0</math>
 +
 
 +
<font color="#ff0000"><span style="font-size: 17px;">'''&nbsp; &nbsp; &nbsp; &nbsp; &nbsp; &nbsp; &nbsp; &nbsp; &nbsp; &nbsp; &nbsp; &nbsp; &nbsp; &nbsp; &nbsp; &nbsp;&nbsp;<math>\text{2. } \begin{bmatrix}
 +
2 \left(x_{1}-1 \right) & -1
 +
\end{bmatrix} +\mu \begin{bmatrix}
 +
1 & 1
 +
\end{bmatrix} = \begin{bmatrix}
 +
0 & 0
 +
\end{bmatrix}</math>'''</span></font>
 +
 
 +
<font color="#ff0000">'''&nbsp; &nbsp; &nbsp; &nbsp; &nbsp; &nbsp; &nbsp; &nbsp; &nbsp; &nbsp; &nbsp; &nbsp; &nbsp; &nbsp; &nbsp; &nbsp; &nbsp; &nbsp; &nbsp; &nbsp;&nbsp;<math>\text{3. } \mu \left( x_{1}+x_{2}-2 \right)=0</math><br>'''</font>
 +
 
 +
<math>\text{From 2, } \mu=1 \text{, and } 2\left ( x_{1}-1 \right )=-1</math><br>
 +
 
 +
<math>\text{From 3, } \left( x_{1}+x_{2} \right) =2</math><font face="serif">''<br>''</font>
 +
 
 +
<math>\text{From above two equations, we obtain a candidate point for the minimizer } x^{*}=\begin{bmatrix}
 +
1/2\\
 +
3/2
 +
\end{bmatrix}</math>
 +
 
 +
<span class="texhtml">Check for SOSC:</span><br>
 +
 
 +
<math>L \left ( x^{*}, \mu ^{*}\right )= F\left ( x^{*}\right )+ \mu ^{*} G\left ( x^{*}\right )=\begin{bmatrix} 2 & 0 \\ 0 & 0 \end{bmatrix}</math>
 +
 
 +
<span class="texhtml">Because μ<sup> * </sup> &gt; 0''&nbsp;'',''w'''e&nbsp;'''h'''a'''v'''e'''''</span>'''''<br> ''' ''
 +
 
 +
<math>\tilde{T}\left( x^{* },\mu^{*} \right)= \left \{ y: \begin{bmatrix}
 +
1 & 1
 +
\end{bmatrix}y=0  \right \} = \left \{ y: =\begin{bmatrix}
 +
a & -a
 +
\end{bmatrix}:a\in\Re  \right \}</math>
 +
 
 +
<math>\text{Hence } y^{T}L\left ( x^{\ast },\mu ^{\ast } \right )y=\begin{bmatrix}
 +
a & -a
 +
\end{bmatrix}\begin{bmatrix}
 +
2 & 0\\
 +
0 & 0
 +
\end{bmatrix}\begin{bmatrix}
 +
a \\
 +
-a
 +
\end{bmatrix}=2a^{2}>0</math><br>
 +
 
 +
<math>\therefore x^{*} \text{ satisfies the SOSC for strict local minimizer. }</math>  
  
 
<br>  
 
<br>  
Line 180: Line 309:
 
----
 
----
  
Automatic Control (AC)- Question 3, August 2011<br>Problem 1: &nbsp;https://www.projectrhea.org/rhea/index.php/ECE-QE_AC3-2011_solusion<br>Problem 2: &nbsp;https://www.projectrhea.org/rhea/index.php/ECE-QE_AC3-2011_solusion-2<br>Problem 3: &nbsp;https://www.projectrhea.org/rhea/index.php/ECE-QE_AC3-2011_solusion-3<br>Problem 4: &nbsp;https://www.projectrhea.org/rhea/index.php/ECE-QE_AC3-2011_solusion-4<br>
+
Automatic Control (AC)- Question 3, August 2011  
  
----
+
Go to
  
[[ECE PhD Qualifying Exams|Back to ECE Qualifying Exams (QE) page]]  
+
*Problem 1: [[ECE-QE AC3-2011 solusion-1|solutions and discussions]]
 +
*Problem 2: [[ECE-QE AC3-2011 solusion-2|solutions and discussions]]
 +
*Problem 3: [[ECE-QE AC3-2011 solusion-3|solutions and discussions]]
 +
*Problem 4: [[ECE-QE AC3-2011 solusion-4|solutions and discussions]]
 +
*Problem 5: [[ECE-QE_AC3-2011_solusion-5|solutions and discussions]]
 +
 
 +
----
  
[[Category:ECE]] [[Category:QE]] [[Category:Automatic_Control]] [[Category:Problem_solving]]
+
[[ECE PhD Qualifying Exams|Back to ECE Qualifying Exams (QE) page]]

Latest revision as of 10:11, 13 September 2013


ECE Ph.D. Qualifying Exam in "Automatic Control" (AC)

Question 3, August 2011, Part 5

Part 1,2,3,4,5

 $ \color{blue}\text{5. } \left( \text{20 pts} \right) \text{ Consider the following optimization problem, } $

                            $ \text{optimize} \left(x_{1}-2\right)^{2}+\left(x_{2}-1\right)^{2} $

                        $ \text{subject to } x_{2}- x_{1}^{2}\geq0 $

                                                 $ 2-x_{1}-x_{2}\geq0 $

                                                 $ x_{1}\geq0. $

$ \color{blue} \text{The point } x^{*}=\begin{bmatrix} 0 & 0 \end{bmatrix}^{T} \text{ satisfies the KKT conditions.} $


Theorem:

For the problem:    minimize  $ f \left( x \right) $

                             subject to   $ h \left( x \right) =0 $

                                                $ g \left( x \right) \leq 0 $

The KKT condition (FONC) for local minimizer x *  of f is:

      $ \text{1. } \mu^{*}\geq0 $

     $ \text{2. } Df\left ( x^{*} \right )+\lambda ^{*T}Dh\left ( x^{*} \right )+\mu ^{*T}Dg\left ( x^{*} \right )=0^{T} $

        $ \text{3. } \mu ^{*T}g\left ( x^{*} \right )=0 $

        $ \text{4. } h\left ( x^{*} \right )=0 $

        $ \text{5. } g \left( x^{*} \right) \leq0 $

Definision: Regular point              

         $ x^{*} \text{ satisfy } h\left( x^{*} \right)=0, g\left( x^{*} \right)\leq0 \text{ and let } J\left(x^{*}\right)= \left \{ j:g_{j}\left(x^{*}\right)=0 \right \} $

         $ x^{*}\text{ is regular point if } \nabla h_{i} \left( x^{*} \right), \nabla g_{j} \left( x^{*} \right), 1\leq i\leq m, j\in J \left( x^{*} \right) $

SONC: Suppose that x *  is regular
        $ \text{1. } \mu ^{*}\geq0 \text{, } Df\left ( x^{*} \right )+\lambda ^{*T}Dh\left ( x^{*} \right )+\mu ^{*T}Dg\left ( x^{*} \right )=0^{T} \text{, } \mu ^{*T}g\left ( x^{*} \right )=0 $

        $ \text{2. For all } y\in T\left( x^{*} \right ) \text{, we have } y^{T}L\left ( x^{\ast },\mu ^{\ast }, \lambda ^{\ast }\right )y\geq 0 $

               $ T\left( x^{* } \right)= \left \{ y\in\Re^{n}: Dh\left( x^{*} \right)y=0, Dg_{j}\left( x^{*} \right)y=0, j\in J\left( x^{*} \right) \right \} $

               $ J\left(x^{*}\right)= \left \{ j:g_{j}\left(x^{*}\right)=0 \right \} $

SOSC: There exist a feasible point xthat 

        $ \text{1. } \mu ^{*}\geq0 \text{, } Df\left ( x^{*} \right )+\lambda ^{*T}Dh\left ( x^{*} \right )+\mu ^{*T}Dg\left ( x^{*} \right )=0^{T} \text{, } \mu ^{*T}g\left ( x^{*} \right )=0 $

        $ \text{2. For all } y\in \tilde{T}\left( x^{* }\mu^{*} \right) \text{, we have } y^{T}L\left ( x^{\ast },\mu ^{\ast }, \lambda ^{\ast }\right )y\geq 0 $

               $ \tilde{T}\left( x^{* },\mu^{*} \right)= \left \{ y: Dh\left( x^{*} \right)y=0, Dg_{i}\left( x^{*} \right)y=0, i\in \tilde{J}\left( x^{*},\mu^{*} \right) \right \} $

              $ \tilde{J}\left ( x^{\ast },\mu ^{\ast } \right )= \left \{ i:g_{i}\left ( x^{\ast } \right ) = 0,\mu_{i}^{\ast }> 0\right \} $

Process:

      a. Write down the KKT condition for this probelm

      b. Find all points (and KKT multipliers) satisfying the KKT condition. In each case, determine if the point is regular.

      c. Find all points in part b that also satisfy the SONC.

      d. Find all points in part c that also satisfy the SOSC.

      e. Find all points in part c that are local minimizers.


$ \color{blue}\left( \text{i} \right) \text{Does } x^{*} \text{ satisfy the FONC for minimum or maximum? Where are the KKT multipliers?} $

$ \color{blue}\text{Solution 1:} $

        $ f\left( x \right) = \left(x_{1}-2\right)^{2}+\left(x_{2}-1\right)^{2} $

        $ g_{1}\left( x \right)=x_{1}^{2}-x_{2} $

        $ g_{2}\left( x \right)= x_{1}+x_{2}-2 $

        $ g_{3}\left( x \right)= -x_{1} $

 $ \text{ The problem is to optimize f(x), subject to } g_{1}\leq 0, g_{2}\leq 0, g_{3}\leq 0 $

$ \text{Let } l\left( \mu ,\lambda \right)=\nabla f\left(x \right)+\mu_{1} \nabla g_{1}\left( x \right)+\mu_{2} \nabla g_{2}\left( x \right)+\mu_{3} \nabla g_{3}\left( x \right) $

                      $ =\begin{pmatrix} 2x_{1}-4\\ 2x_{2}-2 \end{pmatrix} +\mu_{1} \begin{pmatrix} 2x_{1}\\ -1 \end{pmatrix}+\mu_{2}+\begin{pmatrix} 1\\ 1 \end{pmatrix}+\mu_{3}+\begin{pmatrix} -1\\ 0 \end{pmatrix} =0 $

$ \mu_{1} g_{1}\left( x \right)+\mu_{2} g_{2}\left( x \right)+\mu_{3} g_{3}\left( x \right) $

            $ = \mu_{1} \left( x_{1}^2-x_{2} \right)+\mu_{2} \left( x_{1}+x_{2}-2 \right)+\mu_{3} \left( -x_{1} \right) =0 $

$ \text{Let } x^{*}=\begin{bmatrix} 0\\ 0 \end{bmatrix} \text{, } $

            $ \left\{\begin{matrix} \nabla l\left( x,\mu \right)=\begin{pmatrix} -4+\mu_{2}-\mu_{3}\\ -2-\mu_{1}-\mu_{2} \end{pmatrix}= \begin{pmatrix} 0 \\ 0\end{pmatrix} \\ -2\mu_{2}=0 \end{matrix}\right. \Rightarrow \left\{\begin{matrix} \mu_{1}=-2\\ \mu_{2}=0\\ \mu_{3}=-4 \end{matrix}\right. $

$ \text{As } \mu^{*}\leq 0, x^{*}\begin{bmatrix} 0\\0 \end{bmatrix} \text{satisfies the FONC for maximum.} $


$ \color{blue}\text{Solution 2:} $

$ \text{ Standard form: optimize} \left(x_{1}-2\right)^{2}+\left(x_{2}-1\right)^{2} $

                                  $ \text{subject to } g_{1}\left( x \right)= x_{1}^{2}-x_{2}\leq0 $

                                                           $ g_{2}\left( x \right)= x_{1}+x_{2}-2\leq0 $

                                                           $ g_{3}\left( x \right)= -x_{1}\leq0 $

$ \text{KKT condition: (1) } Dl\left( \mu ,\lambda \right)=Df\left(x \right)+\mu_{1}Dg_{1}\left( x \right)+\mu_{2}Dg_{2}\left( x \right)+\mu_{3}Dg_{3}\left( x \right) $

                                             $ =\left [ 2x_{1}-4+2\mu_{1}x_{1}+\mu_{2}-\mu_{3}, 2x_{2}-2-\mu_{1}+\mu_{2} \right ]=0 $                                            $ \left ( 2 \right ) \mu^{T}g\left ( x \right )=0 \Rightarrow \mu_{1}\left ( x_{1}^2-x_{2} \right )+\mu_{2}\left ( x_{1}+x_{2}-2 \right ) - \mu_{3}x_{1}=0 $

                                 $ \left ( 3 \right ) \mu_{1},\mu_{2},\mu_{3}\geq 0 \text{ for minimizer} $

                                        $ \mu_{1},\mu_{2},\mu_{3}\leq 0 \text{ for maximizer} $

                                        $ \text{where } \mu^{*}=\begin{bmatrix} \mu_{1}\\ \mu_{2}\\ \mu_{3} \end{bmatrix} \text{ are the KKT multiplier.} $

$ \text{For } x^{*}=\begin{bmatrix} 0\\ 0 \end{bmatrix} \text{, } $       $ \left\{\begin{matrix} \nabla l\left( x,\mu \right)=\begin{pmatrix} -4+\mu_{2}-\mu_{3}\\ -2-\mu_{1}+\mu_{2} \end{pmatrix}=\begin{pmatrix} 0\\0 \end{pmatrix}\\ -2\mu_{2}=0 \end{matrix}\right. \Rightarrow \left\{\begin{matrix} \mu_{1}=-2\\ \mu_{2}=0\\ \mu_{3}=-4 \end{matrix}\right. $

$ \therefore x^{*}=\begin{bmatrix} 0 \\ 0 \end{bmatrix} \text{ satisfy FONC for maximum} $


$ \color{blue}\left( \text{ii} \right) \text{Does } x^{*} \text{ satisfy SOSC? Carefully justify your answer.} $

$ \color{blue}\text{Solution 1:} $

$ L\left ( x^{*},\mu^{*} \right )= \nabla l \left( x^{*},\mu^{*} \right)= \begin{pmatrix} 2&0 \\ 0&2 \end{pmatrix}-2\begin{pmatrix} 2&0 \\ 0&0 \end{pmatrix} = \begin{pmatrix} -2&0 \\ 0&2 \end{pmatrix} $

$ \tilde{T}\left( x^{* }\mu^{*} \right) : \left\{ \begin{matrix} y^{T}\binom{0}{-1} =0 \\ y^{T}\binom{-1}{0} =0 \end{matrix} \right. \Rightarrow \tilde{T}\left( x^{* }\mu^{*} \right)= \left \{ \binom{0}{0} \right \} $

SOSC is trivially satisfied.

$ \color{red} \text{This solution misunderstood the range of } y \text{ for SOSC condition } y^{T}L\left ( x^{\ast },\mu ^{\ast } \right )y\geq 0 $

$ \color{red} y\in \tilde{T}\left( x^{* }\mu^{*} \right) \text{, where } \tilde{T}\left( x^{* },\mu^{*} \right)= \left \{ y:Dg_{i}\left( x^{*} \right)y=0, i\in \tilde{J}\left( x^{*},\mu^{*} \right) \right \} $

$ \color{red} \tilde{J}\left ( x^{\ast },\mu ^{\ast } \right ) \text{ has constrain for } \mu ^{\ast } \text{, } \tilde{J}\left ( x^{\ast },\mu ^{\ast } \right )= \left \{ i:g_{i}\left ( x^{\ast } \right ) = 0,\mu_{i}^{\ast }> 0\right \} $

$ \color{red} \tilde{J}\left ( x^{\ast },\mu ^{\ast } \right ) \subset J\left(x^{*}\right) $,    $ \color{red} T\left( x^{* } \right) \subset \tilde{T}\left( x^{* },\mu^{*} \right) $


$ \color{blue}\text{Solution 2:} $

$ L\left ( x_{1},\mu \right )= D^{2} l \left ( x _{1},\mu \right )= \begin{bmatrix} 2+2\mu_{1} & 0 \\ 0 & 2 \end{bmatrix} $

                   $ \text{for point } x^{*}=\begin{bmatrix} 0 \\ 0 \end{bmatrix} \text{, we get } \mu_{1}=-2 \text{ from KKT condition.} $

           $ \therefore L \left ( x^{*}, \mu ^{*}\right )=\begin{bmatrix} -2 & 0 \\ 0 & 2 \end{bmatrix} $

$ \tilde{T}\left( x^{* },\mu^{*} \right)= \left \{ y:Dg_{i}\left( x^{*} \right)y=0, i\in \tilde{J}\left( x^{*},\mu^{*} \right) \right \} $

             $ \tilde{J}\left ( x^{\ast },\mu ^{\ast } \right )= \left \{ i:g_{i}\left ( x^{\ast } \right ) = 0,\mu_{i}^{\ast }> 0\right \} $     $ \therefore i= 2 $

             $ \therefore \tilde{T}\left ( x^{\ast },\mu ^{\ast } \right )= \left \{ y:\left [ 1,1 \right ]y= 0 \right \}= \left \{ y:y_{1}= -y_{2} \right \} $

$ y^{T}L\left ( x^{\ast },\mu ^{\ast } \right )y=\begin{bmatrix} y_{1}& y_{2} \end{bmatrix}\begin{bmatrix} -2 & 0\\ 0 & 2 \end{bmatrix} \begin{bmatrix} y_{1}\\ y_{2} \end{bmatrix} \geqslant 0 $

                    $ -2y_{1}^{2}+2y_{2}^{2}\geqslant 0\cdots \left ( 1 \right ) $

                    for y1 = - y2,  (1) is always satisfied.

$ \therefore \text{For all } y\in \tilde{T}\left( x^{* },\mu^{*} \right) \text{, we have } y^{T}L\left ( x^{\ast },\mu ^{\ast } \right )y\geq 0 $

$ \therefore \text{point } x^{*} \text{satisfy the SOSC} $



$ \color{blue}\text{Related Problem: } $

                  minimize    − x2 + (x1 − 1)2 − 2

                  $ \text{subject to } x_{1}+x_{2}\leq2 $

$ \text{KKT condition: 1. } \mu\geq0 $

                                $ \text{2. } \begin{bmatrix} 2 \left(x_{1}-1 \right) & -1 \end{bmatrix} +\mu \begin{bmatrix} 1 & 1 \end{bmatrix} = \begin{bmatrix} 0 & 0 \end{bmatrix} $

                                        $ \text{3. } \mu \left( x_{1}+x_{2}-2 \right)=0 $

$ \text{From 2, } \mu=1 \text{, and } 2\left ( x_{1}-1 \right )=-1 $

$ \text{From 3, } \left( x_{1}+x_{2} \right) =2 $

$ \text{From above two equations, we obtain a candidate point for the minimizer } x^{*}=\begin{bmatrix} 1/2\\ 3/2 \end{bmatrix} $

Check for SOSC:

$ L \left ( x^{*}, \mu ^{*}\right )= F\left ( x^{*}\right )+ \mu ^{*} G\left ( x^{*}\right )=\begin{bmatrix} 2 & 0 \\ 0 & 0 \end{bmatrix} $

Because μ * > 0 ,whave

$ \tilde{T}\left( x^{* },\mu^{*} \right)= \left \{ y: \begin{bmatrix} 1 & 1 \end{bmatrix}y=0 \right \} = \left \{ y: =\begin{bmatrix} a & -a \end{bmatrix}:a\in\Re \right \} $

$ \text{Hence } y^{T}L\left ( x^{\ast },\mu ^{\ast } \right )y=\begin{bmatrix} a & -a \end{bmatrix}\begin{bmatrix} 2 & 0\\ 0 & 0 \end{bmatrix}\begin{bmatrix} a \\ -a \end{bmatrix}=2a^{2}>0 $

$ \therefore x^{*} \text{ satisfies the SOSC for strict local minimizer. } $



Automatic Control (AC)- Question 3, August 2011

Go to


Back to ECE Qualifying Exams (QE) page

Alumni Liaison

Meet a recent graduate heading to Sweden for a Postdoctorate.

Christine Berkesch